Die Kontraktion des Fermionenfeldes in der 1+1-dimensionalen masselosen QED

Meine Frage stammt aus dem Lehrbuch von Peskin & Schroeder, dem Integral (19.26):

D 2 k ( 2 π ) 2 e ich k ( j z ) ich k k 2 = ∂̸ ( ich 4 π Protokoll ( j z ) 2 )

Frage: Wie leitet man die Formel von der linken Seite auf die rechte Seite ab?

Betrachtet man die Identität (3.117) und set M = 0 , Ich habe

D 2 k ( 2 π ) 2 ich k γ k 2 e ich k ( j z ) = ich ∂̸ ( D R ( j z ) )
Hier
D R ( j z ) = D 2 k ( 2 π ) 2 ich k 2 e ich k ( j z )
der 2-Vektor: k μ = ( k 0 , k 1 ) und wegen der masselosen Bedingung: ( k 0 ) 2 = ( k 1 ) 2 . Satz κ k 1 .daher habe ich
+ D k 1 ( 2 π ) [ 1 2 k 0 e ich [ k 0 ( j z ) 0 k 1 ( j z ) 1 ] + 1 2 k 0 e ich [ k 0 ( j z ) 0 k 1 ( j z ) 1 ] ] = ich 4 π   2 + Sünde ( κ ( j z ) 0 ) κ e ich κ ( j z ) 1 D κ

Aber ich konnte den Log-Term nicht aus der obigen Formel bekommen.

ANMERKUNG Ich habe eine verwandte Antwort gefunden Ein vierdimensionales Integral in Peskin & Schroeder

Man kann den On-Shell-Zustand hier nicht verwenden, da die Komponenten von k μ sind unabhängig.

Antworten (1)

Ich habe festgestellt, dass eine Fourier-Transformation diese Frage beantworten kann. Lassen Sie mich den euklidischen Raum durch die Wick-Rotation bearbeiten: k 0 ich k 2 , somit

D 2 k ( 2 π ) 2 ich k k 2 e ich k ( j z ) D k 1 D k 2 ( 2 π ) 2 ich k | k | 2 e ich k ( j z ) Satz: j z := R = ich R 1 ( 2 π ) 2 1 k 1 2 + k 2 2 e ich ( k 1 R 1 + k 2 R 2 ) D k 1 D k 2 = ich R ( 1 ( 2 π ) 2 [ 2 π Protokoll | R | ] ) = ich R ( 1 4 π Protokoll ( j z ) 2 )
hier ist der blaue Teil ein Fourier-Integral in 2-D, diese Fourier-Transformation ist in der Green-Funktion des Laplace-Operators im 2-dimensionalen Raum vorhanden.

ich an der Spitze von RHS wird die erste Zeile weggelassen. Und Protokoll | R | = Protokoll | j z | 2 1 / 2 = Protokoll ( ( j z ) 2 ) 1 / 2 . Die letzte Zeile ist also
ich R ( 1 4 π Protokoll ( ( j z ) 2 ) ) .
Dann ist es in Übereinstimmung mit einer anderen Methode . Die zweite Zeile von (19.26) im Lehrbuch ist falsch gedruckt.